LSAT and Law School Admissions Forum

Get expert LSAT preparation and law school admissions advice from PowerScore Test Preparation.

 Administrator
PowerScore Staff
  • PowerScore Staff
  • Posts: 8919
  • Joined: Feb 02, 2011
|
#33115
Complete Question Explanation

Strengthen—PR, SN. The correct answer choice is (A)

The wording of this stimulus is quite difficult for a question that begins a Logical Reasoning section. Not only does it contain an unusual conclusion, including multiple necessary conditions, but it also has a necessary condition modified by “unless.” Additionally, the construction of the conditional terms is designed to confuse you by including language implying nested conditional statements, such as “sufficiently certain” and “justify imposing.”

The evidence described in the stimulus suggests that the local auto repair shop is responsible for pollution that recently entered a river. The problem is, the penalty for this type of pollution is so severe that the authorities conclude they cannot be justified in imposing it unless the evidence against the auto repair shop is stronger, or the shop admits responsibility.

Since the conditional relationship in the conclusion uses the word “unless,” it should be diagrammed using the Unless Equation. The Unless Equation is a simple, mechanical way to diagram conditional relationships in which the necessary condition is modified by “unless,” “except,” “until,” or “without.” The mechanical process is to make whichever term is modified by one of these four words the necessary condition. The remaining term is negated, and becomes the sufficient condition. Here, you should diagram the relationship in the conclusion as:
  • ..... ..... ..... ..... ..... ..... ..... ..... ..... ..... ..... stronger evidence discovered

    sufficiently certain to justify imposing penalty ..... :arrow: ..... ..... ..... OR

    ..... ..... ..... ..... ..... ..... ..... ..... ..... ..... ..... perpetrator admits responsibility
This conditional relationship results from the application of some rule, not expressly stated in the stimulus, to the premises. The absence of the rule from the stimulus is a logical flaw. In this Strengthen question, your prephrase is that the correct answer choice will provide the rule that supports the conclusion’s conditional statement, thereby linking together the premises and the conclusion.

Answer choice (A): This is the correct answer choice. In the stimulus, the conclusion was supported by the opinion that “the penalty for this type of pollution is so severe...” This description implies that the penalty for the pollution offense alleged is more severe than some other penalties, and that the more severe the penalty, the stronger the evidence must be to justify imposing it. This answer choice provides the rule missing from the stimulus, thus bridging the premises to the conclusion.

Answer choice (B): This answer choice provides a rule regarding how severe penalties should be. In the stimulus, the severity of the penalty was not at issue. Instead, the issue was the strength of the evidence required when the penalty prescribed for a violation is so severe.

Answer choice (C): As with answer choice (B), this choice provides a rule for use in setting the severity of the penalty for an infraction, while the stimulus discussed the evidence required to justify the imposition of such a severe penalty.

Answer choice (D): This answer choice provides a rule that assists in predicting the likelihood that a person will admit responsibility for an offense, rather than one that requires stronger evidence when imposing stronger penalties.

Answer choice (E): This answer choice is similar to answer choices (B) and (C), in that it presents a rule that would apply when setting the penalty for an offense, instead of a rule that applies to the determination of whether the imposition of the penalty is justified.

Get the most out of your LSAT Prep Plus subscription.

Analyze and track your performance with our Testing and Analytics Package.